Last visit was: 11 May 2024, 03:28 It is currently 11 May 2024, 03:28

Close
GMAT Club Daily Prep
Thank you for using the timer - this advanced tool can estimate your performance and suggest more practice questions. We have subscribed you to Daily Prep Questions via email.

Customized
for You

we will pick new questions that match your level based on your Timer History

Track
Your Progress

every week, we’ll send you an estimated GMAT score based on your performance

Practice
Pays

we will pick new questions that match your level based on your Timer History
Not interested in getting valuable practice questions and articles delivered to your email? No problem, unsubscribe here.
Close
Request Expert Reply
Confirm Cancel
SORT BY:
Date
Tags:
Show Tags
Hide Tags
Math Expert
Joined: 02 Sep 2009
Posts: 93155
Own Kudos [?]: 622776 [4]
Given Kudos: 81832
Send PM
Retired Moderator
Joined: 18 May 2019
Posts: 785
Own Kudos [?]: 1041 [3]
Given Kudos: 101
Send PM
Director
Director
Joined: 30 Sep 2017
Posts: 956
Own Kudos [?]: 1258 [1]
Given Kudos: 402
GMAT 1: 720 Q49 V40
GPA: 3.8
Send PM
SVP
SVP
Joined: 24 Nov 2016
Posts: 1720
Own Kudos [?]: 1346 [1]
Given Kudos: 607
Location: United States
Send PM
Re: Unless they are used as strictly temporary measures, rent-control ordi [#permalink]
1
Kudos
Quote:
Unless they are used as strictly temporary measures, rent-control ordinances (municipal regulations placing limits on rent increases) have several negative effects for renters. One of these is that the controls will bring about a shortage of rental units. This disadvantage for renters occurs over the long run, but the advantage—smaller rent increases—occurs immediately. In many municipalities, specifically in all those where tenants of rent-control units have a secure hold on political power and can get rent-control ordinances enacted or repealed, it is invariably the desire for short-term gain that guides those tenants in the exercise of that power.

If the statements above are true, which one of the following can be properly inferred from them?


(A) It is impossible for landlords to raise rents when rent controls are in effect.

(B) In many municipalities rent-control ordinances are repealed as soon as shortages of rental unites arise.

(C) The only negative effect of tent control for renters is that it brings about a shortage of rental units.

(D) In many municipalities there is now, or eventually will be, a shortage of rental units.

(E) In the long term, a shortage of rental units will raise rents substantially.


WHAT CHOICE "MUST BE TRUE"
(A) munis place "limits on rent raise", thus landlords are allowed to raise rents;
(B) cannot be inferred;
(C) there are "several negative effects";
(E) cannot be inferred;

Ans (D) yes, now or in the future, there will be a shortage of units.
VP
VP
Joined: 27 Feb 2017
Posts: 1487
Own Kudos [?]: 2305 [2]
Given Kudos: 114
Location: United States (WA)
GMAT 1: 760 Q50 V42
GMAT 2: 760 Q50 V42
GRE 1: Q169 V168

GRE 2: Q170 V170
Send PM
Re: Unless they are used as strictly temporary measures, rent-control ordi [#permalink]
2
Kudos
It took me 2 minutes to decide on my choice of (D).

(D) can be strongly supported by the texts above. "In many municipalities, ..., it is invariably the desire for short-term gain that guides those tenants in the exercise of that power." And the "short-term gain" is to implement rent control. It results in, among other things, a shortage of rental units over the long run. So, depending on the starting date of the rent control, in many municipalities there is now, or eventually will be, a shortage of rental units.

Other choices
(A) Contradicts the info that rent control still allow rent increase, albeit limited ones.
(B) Such ordinance will result in shortage of rental units, but shortage of rental units won't necessarily repeal the ordinance.
(C) Shortage is only one negative effect.
(E) In the long run, there will be a shortage of rental units. But as long as the rent control is in place, rental price upward is limited.
Manager
Manager
Joined: 14 Sep 2019
Posts: 223
Own Kudos [?]: 138 [1]
Given Kudos: 31
Send PM
Re: Unless they are used as strictly temporary measures, rent-control ordi [#permalink]
1
Kudos
(D) In many municipalities there is now, or eventually will be, a shortage of rental units.

We can assume that rental units may decrease.
Answer:D
CEO
CEO
Joined: 07 Mar 2019
Posts: 2568
Own Kudos [?]: 1826 [0]
Given Kudos: 763
Location: India
WE:Sales (Energy and Utilities)
Send PM
Re: Unless they are used as strictly temporary measures, rent-control ordi [#permalink]
Unless they are used as strictly temporary measures, rent-control ordinances (municipal regulations placing limits on rent increases) have several negative effects for renters. One of these is that the controls will bring about a shortage of rental units. This disadvantage for renters occurs over the long run, but the advantage—smaller rent increases—occurs immediately. In many municipalities, specifically in all those where tenants of rent-control units have a secure hold on political power and can get rent-control ordinances enacted or repealed, it is invariably the desire for short-term gain that guides those tenants in the exercise of that power.

If the statements above are true, which one of the following can be properly inferred from them?

(A) It is impossible for landlords to raise rents when rent controls are in effect.

(B) In many municipalities rent-control ordinances are repealed as soon as shortages of rental unites arise.

(C) The only negative effect of tent control for renters is that it brings about a shortage of rental units.

(D) In many municipalities there is now, or eventually will be, a shortage of rental units.

(E) In the long term, a shortage of rental units will raise rents substantially.

B looks good since for short term people would be willingly take advantage of the ordinance i.e. people, naturally, would use benefits before the moment things balance out.

IMO Answer B.
Current Student
Joined: 24 Jul 2019
Posts: 207
Own Kudos [?]: 365 [0]
Given Kudos: 162
GMAT 1: 730 Q46 V45
GPA: 3.9
Send PM
Unless they are used as strictly temporary measures, rent-control ordi [#permalink]
This is a very good inference question and it preys upon those who take their 'real world' knowledge into the GMAT world.
I initially settled for (E) but was not confident with that answer, after reading the prompt for a second time I knew that (E) is the trap answer.

A general theme that you will see on inference questions is that the right answer is formulated as vaguely, and with no absolutes, as possible.
'... eventually will be' is your trigger word, would the sentence be phrased as 'There WILL be ...' it would be straight wrong.

Now, (E) displays one of the most favorite traps of the test maker. It's just a statement everybody, no matter if preparing for the GMAT or not, agrees upon. A shortgage of rental units will lead to a raise in rents, who could disagree with that? It's perfectly reasoned isn't it?
Well, it indeed is! But we are on the LSAT/GMAT inference section and nobody cares about what is common sense - we only care about information that is given in the prompt:

"Unless they are used as strictly temporary measures, rent-control ordinances (municipal regulations placing limits on rent increases) have several negative effects for renters. One of these is that the controls will bring about a shortage of rental units. This disadvantage for renters occurs over the long run"

Is there anywhere mentioned that the decrease in supply will raise the rents? No! Discard (E) and pick (D). :)
Manager
Manager
Joined: 04 Aug 2019
Posts: 63
Own Kudos [?]: 70 [0]
Given Kudos: 746
Location: Viet Nam
Concentration: Organizational Behavior, Strategy
Schools: Desautels '23
GMAT 1: 740 Q49 V42
WE:Research (Other)
Send PM
Re: Unless they are used as strictly temporary measures, rent-control ordi [#permalink]
Bunuel wrote:

Competition Mode Question



Unless they are used as strictly temporary measures, rent-control ordinances (municipal regulations placing limits on rent increases) have several negative effects for renters. One of these is that the controls will bring about a shortage of rental units. This disadvantage for renters occurs over the long run, but the advantage—smaller rent increases—occurs immediately. In many municipalities, specifically in all those where tenants of rent-control units have a secure hold on political power and can get rent-control ordinances enacted or repealed, it is invariably the desire for short-term gain that guides those tenants in the exercise of that power.

If the statements above are true, which one of the following can be properly inferred from them?


(A) It is impossible for landlords to raise rents when rent controls are in effect.

(B) In many municipalities rent-control ordinances are repealed as soon as shortages of rental unites arise.

(C) The only negative effect of tent control for renters is that it brings about a shortage of rental units.

(D) In many municipalities there is now, or eventually will be, a shortage of rental units.

(E) In the long term, a shortage of rental units will raise rents substantially.


GOAL: Find a statement that MUST BE TRUE if the statements given are true. So, we need to do a POE here, trying to be extremely LITERAL when reading the argument and the answer choices, noticing words that may trick us: "ONE of these", "many", etc.

Now the POE:

(A) It is impossible for landlords to raise rents when rent controls are in effect.
We know that when rent controls are in effect, limits are placed on RENT INCREASES. This does not mean that landlords CAN NOT RAISE rents, but only means that the increases are limited. Eliminate (A).

(B) In many municipalities rent-control ordinances are repealed as soon as shortages of rental unites arise.
First, the argument only says that the enactment of the ordinance will lead to a shortage of rental units in the long-run. HOW do we know what will happen after the shortage, if it happens??? Moreover, if a shortage of rental units occurs, it may be caused by many factors, so we CAN NOT be sure that the tenants will power will REPEAL the ordinances to deal with the shortage at their favor.
Second, "as soon as", which means that "immediately after", is problematic. We CAN NOT be sure that the repeal of rent-control will happen immediately after shortages of rental units arise. Eliminate (B).

(C) The only negative effect of tent control for renters is that it brings about a shortage of rental units.
So this is why we need to be extremely LITERAL when solving CR questions, especially Inference/Must be true ones. The argument says that "ONE OF THESE [disadvantages] is that the controls will bring about a shortage of rental units" rather than that a shortage of rental units is the ONLY disadvantage/negative effect of the rent-control ordinances. Eliminate (C).

(D) In many municipalities there is now, or eventually will be, a shortage of rental units.
Because the tenants in many municipalities who possess power will INVARIABLY exercise that power for SHORT-TERM GAINS, they will want to ENACT the rent-control ordinances so that rent increases will be smaller at their favor (after all, they are the ones who have to pay for the rents). Here we not that the powerful tenants will have the ordinances enacted. Hence, according to the argument, the enactment of the rent-control ordinance will lead to a shortage of rental units OVER THE LONG RUN (so it must be true that the shortage happens now OR in the long-run.). Keep (D)

(E) In the long term, a shortage of rental units will raise rents substantially.
Again, we MUST NOT automatically assume the working of supply-demand law here. The argument only says that the enactment of the ordinance will lead to a shortage of rental units in the long-run. What will happen AFTER that, we don't know. We CAN NOT be sure that AFTER the shortage a decrease in rents will happen . Also, the adjective "SUBSTANTIAL" is problematic, HOW can we know that the decrease in rents, if it happens, will be SUBSTANTIAL??? Eliminate (E)

--> (D) is correct.
User avatar
Non-Human User
Joined: 01 Oct 2013
Posts: 17295
Own Kudos [?]: 850 [0]
Given Kudos: 0
Send PM
Re: Unless they are used as strictly temporary measures, rent-control ordi [#permalink]
Hello from the GMAT Club VerbalBot!

Thanks to another GMAT Club member, I have just discovered this valuable topic, yet it had no discussion for over a year. I am now bumping it up - doing my job. I think you may find it valuable (esp those replies with Kudos).

Want to see all other topics I dig out? Follow me (click follow button on profile). You will receive a summary of all topics I bump in your profile area as well as via email.
GMAT Club Bot
Re: Unless they are used as strictly temporary measures, rent-control ordi [#permalink]
Moderators:
GMAT Club Verbal Expert
6925 posts
GMAT Club Verbal Expert
238 posts
CR Forum Moderator
832 posts

Powered by phpBB © phpBB Group | Emoji artwork provided by EmojiOne